lark manufacturing

This topic has expert replies
Master | Next Rank: 500 Posts
Posts: 234
Joined: Fri Oct 01, 2010 7:28 pm
Location: chennai
Thanked: 5 times
Followed by:4 members

lark manufacturing

by pappueshwar » Wed Mar 21, 2012 10:14 am
Lark Manufacturing Company initiated a voluntary Quality Circles program for machine
operators. Independent surveys of employee attitudes indicated that the machine operators
participating in the program were less satisfied with their work situations after two years of the
program's existence than they were at the program's start. Obviously, any workers who
participate in a Quality Circles program will, as a result, become less satisfied with their jobs.

Each of the following, if true, would weaken the conclusion drawn above EXCEPT:

(A) The second survey occurred during a period of recession when rumors of cutbacks and
layoffs at Lark Manufacturing were plentiful .
(B) The surveys also showed that those Lark machine operators who neither participated in
Quality Circles nor knew anyone who did so reported the same degree of lessened
satisfaction with their work situations as did the Lark machine operators who participated
in Quality Circles.
(C) While participating in Quality Circles at Lark Manufacturing, machine operators exhibited
two of the primary indicators of improved job satisfaction: increased productivity and
decreased absenteeism.
(D) Several workers at Lark Manufacturing who had participated in Quality Circles while
employed at other companies reported that, while participating in Quality Circles in their
previous companies, their work satisfaction had increased.
(E) The machine operators who participated in Quality Circles reported that, when the program
started, they felt that participation might improve their work situations.

OA IS E. DID NT UNDERSTAND THE QUESTION. PLS ASSIST

Junior | Next Rank: 30 Posts
Posts: 26
Joined: Thu Jan 12, 2012 3:33 pm
Thanked: 10 times
GMAT Score:760

by yesman238 » Thu Mar 22, 2012 7:52 am
Here are my thoughts:

A - Weakens the conclusion because it offers another reason for the survey results other than decreased satisfaction (cutbacks and layoffs)

B - Weakens the conclusion because it tackles the assumption that correlation leads to causation (this option basically says that the lessened satisfaction is not due to the program)

C - Weakens because it invalidate the results of the survey, or viewed another way, it offers another reason for the survey results other than participation in the program. This option is saying that the survey should actually show a increase, but since it doesn't, either (1) the results are untrue, or (2) something else is causing the results.

D - Basically the same reason as C.

E - It doesn't weaken the conclusion because it only talks about how the workers felt about the program. It doesn't reference the conclusion at all.

Let me know if it is still unclear.

Senior | Next Rank: 100 Posts
Posts: 83
Joined: Fri May 06, 2011 9:08 am
Thanked: 5 times
GMAT Score:610

by scholardream » Thu Mar 22, 2012 5:00 pm
Thanks pappueshwar, you always post challenging questions. They did help me a lot to improve my CR and help me sleep well at night because I was exhaust thinking to solve them!
I have my approach below:
Premise 1: LMC initiated QC.
Premise 2: Surveys: Satisfaction decreases over 2 years
Conclusion: ANY workers in QC -> LESS satisfied
Question: weaken the conclusion: Participating in QC improve or not decrease Satisfaction. So I'll look for answer provide reasons to improve or at least don't decrease the satisfaction of employees.
(A) Cutbacks & layoffs -> staffs feel dissatificated. Of course! It provides another reason to explain the decrease of satisfaction, so QC is not the reason. WEAKEN
(B) QC has no impact on level of satisfaction. WEAKEN
(C) QC improves satisfaction. WEAKEN
(D) Other workers from other companies (any worker) feel satisfaction increased. WEAKEN
(E) Before joined QC, they expected QC could help improve their work situations. NO information to conclude that participating in QC could improve or decrease their participation. The (E) is just a decription!

Could you give your idea ?

Master | Next Rank: 500 Posts
Posts: 234
Joined: Fri Oct 01, 2010 7:28 pm
Location: chennai
Thanked: 5 times
Followed by:4 members

by pappueshwar » Fri Mar 23, 2012 1:23 am
hi scholar dream,

thanks that my posts are helping you. well regarding this CR i got a a complete idea after reading your way of solving. in the first attempt i just could not understand the argument at all.

thanks once again. its clear now